Martingale and indicator function












0












$begingroup$


For $tge 0$ define the stochastic process
$$Y_t:[0,1]rightarrowmathbb{R},quad Y_t(x)=begin{cases}0,&text{if }t-xnotinmathbb{Q}\
1,&text{if }t-xinmathbb{Q}
end{cases}$$

on the filtration $(F_t)_{tge0}$, $F_t=mathcal{B}(mathbb{R})$ for all $tge 0$. I want to show, that this is a martingale.



Of course $Y_tle1$, such that $Y_t$ is integrable. For $s,tinmathbb{Q}_+$ I find $Y_t=Y_s$. So we have the martingale property for this case. But how do you find the solution for the other cases? And what can you say about the continuity of the paths of this process? Thanks in advance for any help!










share|cite|improve this question











$endgroup$












  • $begingroup$
    For every $t$, $P(Y_t=0)=1$ and $Y_t$ is measurable with respect to $mathcal B(mathbb R)$ hence, for every $s<t$, $E(Y_tmidmathcal F_s)=Y_t=0=Y_s$ almost surely, which shows $(Y_t)$ is an $(mathcal F_t)$-martingale.
    $endgroup$
    – Did
    Jan 8 at 22:30


















0












$begingroup$


For $tge 0$ define the stochastic process
$$Y_t:[0,1]rightarrowmathbb{R},quad Y_t(x)=begin{cases}0,&text{if }t-xnotinmathbb{Q}\
1,&text{if }t-xinmathbb{Q}
end{cases}$$

on the filtration $(F_t)_{tge0}$, $F_t=mathcal{B}(mathbb{R})$ for all $tge 0$. I want to show, that this is a martingale.



Of course $Y_tle1$, such that $Y_t$ is integrable. For $s,tinmathbb{Q}_+$ I find $Y_t=Y_s$. So we have the martingale property for this case. But how do you find the solution for the other cases? And what can you say about the continuity of the paths of this process? Thanks in advance for any help!










share|cite|improve this question











$endgroup$












  • $begingroup$
    For every $t$, $P(Y_t=0)=1$ and $Y_t$ is measurable with respect to $mathcal B(mathbb R)$ hence, for every $s<t$, $E(Y_tmidmathcal F_s)=Y_t=0=Y_s$ almost surely, which shows $(Y_t)$ is an $(mathcal F_t)$-martingale.
    $endgroup$
    – Did
    Jan 8 at 22:30
















0












0








0


1



$begingroup$


For $tge 0$ define the stochastic process
$$Y_t:[0,1]rightarrowmathbb{R},quad Y_t(x)=begin{cases}0,&text{if }t-xnotinmathbb{Q}\
1,&text{if }t-xinmathbb{Q}
end{cases}$$

on the filtration $(F_t)_{tge0}$, $F_t=mathcal{B}(mathbb{R})$ for all $tge 0$. I want to show, that this is a martingale.



Of course $Y_tle1$, such that $Y_t$ is integrable. For $s,tinmathbb{Q}_+$ I find $Y_t=Y_s$. So we have the martingale property for this case. But how do you find the solution for the other cases? And what can you say about the continuity of the paths of this process? Thanks in advance for any help!










share|cite|improve this question











$endgroup$




For $tge 0$ define the stochastic process
$$Y_t:[0,1]rightarrowmathbb{R},quad Y_t(x)=begin{cases}0,&text{if }t-xnotinmathbb{Q}\
1,&text{if }t-xinmathbb{Q}
end{cases}$$

on the filtration $(F_t)_{tge0}$, $F_t=mathcal{B}(mathbb{R})$ for all $tge 0$. I want to show, that this is a martingale.



Of course $Y_tle1$, such that $Y_t$ is integrable. For $s,tinmathbb{Q}_+$ I find $Y_t=Y_s$. So we have the martingale property for this case. But how do you find the solution for the other cases? And what can you say about the continuity of the paths of this process? Thanks in advance for any help!







probability-theory stochastic-processes






share|cite|improve this question















share|cite|improve this question













share|cite|improve this question




share|cite|improve this question








edited Jan 8 at 22:24







user628255

















asked Jan 8 at 22:15









user628255user628255

224




224












  • $begingroup$
    For every $t$, $P(Y_t=0)=1$ and $Y_t$ is measurable with respect to $mathcal B(mathbb R)$ hence, for every $s<t$, $E(Y_tmidmathcal F_s)=Y_t=0=Y_s$ almost surely, which shows $(Y_t)$ is an $(mathcal F_t)$-martingale.
    $endgroup$
    – Did
    Jan 8 at 22:30




















  • $begingroup$
    For every $t$, $P(Y_t=0)=1$ and $Y_t$ is measurable with respect to $mathcal B(mathbb R)$ hence, for every $s<t$, $E(Y_tmidmathcal F_s)=Y_t=0=Y_s$ almost surely, which shows $(Y_t)$ is an $(mathcal F_t)$-martingale.
    $endgroup$
    – Did
    Jan 8 at 22:30


















$begingroup$
For every $t$, $P(Y_t=0)=1$ and $Y_t$ is measurable with respect to $mathcal B(mathbb R)$ hence, for every $s<t$, $E(Y_tmidmathcal F_s)=Y_t=0=Y_s$ almost surely, which shows $(Y_t)$ is an $(mathcal F_t)$-martingale.
$endgroup$
– Did
Jan 8 at 22:30






$begingroup$
For every $t$, $P(Y_t=0)=1$ and $Y_t$ is measurable with respect to $mathcal B(mathbb R)$ hence, for every $s<t$, $E(Y_tmidmathcal F_s)=Y_t=0=Y_s$ almost surely, which shows $(Y_t)$ is an $(mathcal F_t)$-martingale.
$endgroup$
– Did
Jan 8 at 22:30












1 Answer
1






active

oldest

votes


















0












$begingroup$

We may write $$
Y_t(x) = 1_{(t+mathbb{Q})cap [0,1]}(x)
$$
and it follows $Y_t$ is $F_t=mathcal{B}(mathbb{R})$ measurable. And also, we can see that $
Y_t(x)= 0
$
holds almost surely, thus giving
$$
E[Y_t|F_s]=E[0|F_s]=0=Y_s
$$
holds almost surely for all $s<t$. This establishes that ${Y_t,F_t}$ is a martingale. Finally, we can observe that for all $xin [0,1]$, the path
$$
tmapsto Y_t(x)=1_{x+mathbb{Q}}(t)
$$
of $Y_t$ is everywhere discontinuous.






share|cite|improve this answer









$endgroup$













    Your Answer





    StackExchange.ifUsing("editor", function () {
    return StackExchange.using("mathjaxEditing", function () {
    StackExchange.MarkdownEditor.creationCallbacks.add(function (editor, postfix) {
    StackExchange.mathjaxEditing.prepareWmdForMathJax(editor, postfix, [["$", "$"], ["\\(","\\)"]]);
    });
    });
    }, "mathjax-editing");

    StackExchange.ready(function() {
    var channelOptions = {
    tags: "".split(" "),
    id: "69"
    };
    initTagRenderer("".split(" "), "".split(" "), channelOptions);

    StackExchange.using("externalEditor", function() {
    // Have to fire editor after snippets, if snippets enabled
    if (StackExchange.settings.snippets.snippetsEnabled) {
    StackExchange.using("snippets", function() {
    createEditor();
    });
    }
    else {
    createEditor();
    }
    });

    function createEditor() {
    StackExchange.prepareEditor({
    heartbeatType: 'answer',
    autoActivateHeartbeat: false,
    convertImagesToLinks: true,
    noModals: true,
    showLowRepImageUploadWarning: true,
    reputationToPostImages: 10,
    bindNavPrevention: true,
    postfix: "",
    imageUploader: {
    brandingHtml: "Powered by u003ca class="icon-imgur-white" href="https://imgur.com/"u003eu003c/au003e",
    contentPolicyHtml: "User contributions licensed under u003ca href="https://creativecommons.org/licenses/by-sa/3.0/"u003ecc by-sa 3.0 with attribution requiredu003c/au003e u003ca href="https://stackoverflow.com/legal/content-policy"u003e(content policy)u003c/au003e",
    allowUrls: true
    },
    noCode: true, onDemand: true,
    discardSelector: ".discard-answer"
    ,immediatelyShowMarkdownHelp:true
    });


    }
    });














    draft saved

    draft discarded


















    StackExchange.ready(
    function () {
    StackExchange.openid.initPostLogin('.new-post-login', 'https%3a%2f%2fmath.stackexchange.com%2fquestions%2f3066805%2fmartingale-and-indicator-function%23new-answer', 'question_page');
    }
    );

    Post as a guest















    Required, but never shown

























    1 Answer
    1






    active

    oldest

    votes








    1 Answer
    1






    active

    oldest

    votes









    active

    oldest

    votes






    active

    oldest

    votes









    0












    $begingroup$

    We may write $$
    Y_t(x) = 1_{(t+mathbb{Q})cap [0,1]}(x)
    $$
    and it follows $Y_t$ is $F_t=mathcal{B}(mathbb{R})$ measurable. And also, we can see that $
    Y_t(x)= 0
    $
    holds almost surely, thus giving
    $$
    E[Y_t|F_s]=E[0|F_s]=0=Y_s
    $$
    holds almost surely for all $s<t$. This establishes that ${Y_t,F_t}$ is a martingale. Finally, we can observe that for all $xin [0,1]$, the path
    $$
    tmapsto Y_t(x)=1_{x+mathbb{Q}}(t)
    $$
    of $Y_t$ is everywhere discontinuous.






    share|cite|improve this answer









    $endgroup$


















      0












      $begingroup$

      We may write $$
      Y_t(x) = 1_{(t+mathbb{Q})cap [0,1]}(x)
      $$
      and it follows $Y_t$ is $F_t=mathcal{B}(mathbb{R})$ measurable. And also, we can see that $
      Y_t(x)= 0
      $
      holds almost surely, thus giving
      $$
      E[Y_t|F_s]=E[0|F_s]=0=Y_s
      $$
      holds almost surely for all $s<t$. This establishes that ${Y_t,F_t}$ is a martingale. Finally, we can observe that for all $xin [0,1]$, the path
      $$
      tmapsto Y_t(x)=1_{x+mathbb{Q}}(t)
      $$
      of $Y_t$ is everywhere discontinuous.






      share|cite|improve this answer









      $endgroup$
















        0












        0








        0





        $begingroup$

        We may write $$
        Y_t(x) = 1_{(t+mathbb{Q})cap [0,1]}(x)
        $$
        and it follows $Y_t$ is $F_t=mathcal{B}(mathbb{R})$ measurable. And also, we can see that $
        Y_t(x)= 0
        $
        holds almost surely, thus giving
        $$
        E[Y_t|F_s]=E[0|F_s]=0=Y_s
        $$
        holds almost surely for all $s<t$. This establishes that ${Y_t,F_t}$ is a martingale. Finally, we can observe that for all $xin [0,1]$, the path
        $$
        tmapsto Y_t(x)=1_{x+mathbb{Q}}(t)
        $$
        of $Y_t$ is everywhere discontinuous.






        share|cite|improve this answer









        $endgroup$



        We may write $$
        Y_t(x) = 1_{(t+mathbb{Q})cap [0,1]}(x)
        $$
        and it follows $Y_t$ is $F_t=mathcal{B}(mathbb{R})$ measurable. And also, we can see that $
        Y_t(x)= 0
        $
        holds almost surely, thus giving
        $$
        E[Y_t|F_s]=E[0|F_s]=0=Y_s
        $$
        holds almost surely for all $s<t$. This establishes that ${Y_t,F_t}$ is a martingale. Finally, we can observe that for all $xin [0,1]$, the path
        $$
        tmapsto Y_t(x)=1_{x+mathbb{Q}}(t)
        $$
        of $Y_t$ is everywhere discontinuous.







        share|cite|improve this answer












        share|cite|improve this answer



        share|cite|improve this answer










        answered Jan 8 at 22:53









        SongSong

        11.1k628




        11.1k628






























            draft saved

            draft discarded




















































            Thanks for contributing an answer to Mathematics Stack Exchange!


            • Please be sure to answer the question. Provide details and share your research!

            But avoid



            • Asking for help, clarification, or responding to other answers.

            • Making statements based on opinion; back them up with references or personal experience.


            Use MathJax to format equations. MathJax reference.


            To learn more, see our tips on writing great answers.




            draft saved


            draft discarded














            StackExchange.ready(
            function () {
            StackExchange.openid.initPostLogin('.new-post-login', 'https%3a%2f%2fmath.stackexchange.com%2fquestions%2f3066805%2fmartingale-and-indicator-function%23new-answer', 'question_page');
            }
            );

            Post as a guest















            Required, but never shown





















































            Required, but never shown














            Required, but never shown












            Required, but never shown







            Required, but never shown

































            Required, but never shown














            Required, but never shown












            Required, but never shown







            Required, but never shown







            Popular posts from this blog

            Can a sorcerer learn a 5th-level spell early by creating spell slots using the Font of Magic feature?

            Does disintegrating a polymorphed enemy still kill it after the 2018 errata?

            A Topological Invariant for $pi_3(U(n))$